Metric area unit conversion with decimal valuesA rectangular field is 0.35 kilometers long and 0.2 kilometerswide. What is the area of the field in square meters? Do notround your answer.mX 5 ?Conversion facts for length1000 millimeters (mm) = 1 meter (m)100 centimeters (cm) = 1 meter (m)10 decimeters (dm) = 1 meter (m)1 decameter (dam) = 10 meters (m)1 hectometer (hm) = 100 meters (m)1 kilometer (km) = 1000 meters (m) I need help with this math problem.

Answers

Answer 1

Given that

The length of the rectangular field is 0.35 km and the width is 0.2 km. And we have to find the area of this field.

Explanation -

The formula for the area of the rectangle is

[tex]Area=length\times breadth[/tex]

On substituting the values we have

[tex]\begin{gathered} Area=0.35\times0.2\text{ km}^2 \\ Area=0.07\text{ km}^2 \end{gathered}[/tex]

Since the conversions are given as

[tex]\begin{gathered} 1\text{ km = 1000 m}^{\text{ }}=10^3\text{ m} \\ 1\text{ km}^2=10^6\text{ m} \end{gathered}[/tex]

So the required answer in sq meters will be

[tex]\begin{gathered} Area=0.07\times10^6\text{ m}^2 \\ Area=7\times10^4\text{ m}^2=70000\text{ m}^2 \end{gathered}[/tex]So the final answer is 70000 sq meters.

Related Questions

I really need help with this question!! How do you solve it?In 1985, they discovered a new sea creature. It was found to have an initial population of 832. Assuming the population was growing at a rate of 2.3% each year, when will the population exceed 1500?

Answers

Solution:

The growth rate is given below as

[tex]r=2.3\%[/tex]

The initial population is

[tex]P_0=832[/tex]

The exponential growth formula is given below as

[tex]\begin{gathered} P=P_0(1+r)^t \\ 832(1+\frac{2.3}{100})^t=1500 \\ 832(1.023)^t=1500 \\ (1.023)^t=\frac{1500}{832} \\ (1.023)^t=1.803 \\ take\text{ ln of both sides} \\ ln(1.023)^t=ln(1.803) \\ tln(1.023)=ln(1.803) \\ t=\frac{ln(1.803)}{ln(1.023)} \\ t=25.9years \\ \end{gathered}[/tex]

Hence,

The final answer is

The population will exceed 1500 in approximately 26 years' time

The mass of an atom or molecule is measured in atomic mass units. Which is greater, a carat or a milligram?

Answers

Notice that:

[tex]\begin{gathered} -23<-21, \\ 8.3<10. \end{gathered}[/tex]

Therefore:

[tex]8.3\times10^{-24}<10\times10^{-24}=10^{-23}<10^{-21}<1.66\times10^{-21}\text{.}[/tex]

Then a carat is greater.

Now, since:

[tex]1atomic\text{ mass unit=8.3}\times10^{-24}carat.[/tex]

Then:

[tex]\frac{1}{8.3}\times10^{24}\text{atomic mass unit=1carat.}[/tex]

Simplifying the above result we get:

[tex]1\text{carat}=1.20\times10^{23}\text{atomic mass units.}[/tex]

Also, since:

[tex]1\text{atomic mass unit=1.66}\times10^{-21}milligram[/tex]

then:

[tex]\frac{1}{1.66}\times10^{21}\text{atomic mass unit=1 milligram.}[/tex]

Simplifying the above result we get:

[tex]1\text{ milligram=6.02}\times10^{20}atomic\text{ mass units.}[/tex]

Answer:

A carat is greater.

[tex]1\text{carat}=1.20\times10^{23}\text{atomic mass units.}[/tex]

and

[tex]1\text{ milligram=6.02}\times10^{20}atomic\text{ mass units.}[/tex]

and

what is the distance of (-23, -14 ) and ( -18,2)

Answers

From the given question,

There are given that two point, (-23, -14) and (-18, 2).

Now,

For finding the distance between two point,

Here use distance formula.

From the distance formula,

[tex]d=\sqrt[]{(x_2-x_1)^2+(y_2-y_1)^2}[/tex]

Where,

[tex]x_1=-23,y_1=-14,x_2=-18,y_2=2[/tex]

Put the all values into the above formula,

Then,

[tex]\begin{gathered} d=\sqrt[]{(x_2-x_1)^2+(y_2-y_1)^2} \\ d=\sqrt[]{(-18_{}-(-23_{})^2+(2_{}-(-14)_{})^2} \\ d=\sqrt[]{(-18_{}+23_{})^2+(2_{}+14_{})^2} \end{gathered}[/tex]

Then,

[tex]\begin{gathered} d=\sqrt[]{(-18_{}+23_{})^2+(2_{}+14_{})^2} \\ d=\sqrt[]{(5_{})^2+(16_{})^2} \\ d=\sqrt[]{25+256} \\ d=\sqrt[]{281} \end{gathered}[/tex]

Hence, the distance of given

The table below shows the population of the state of Washington for several decades starting in1900.Year1900191019201930194019501960Populationin millions0.521.151.371.571.742.392.86If the population is modeled by an exponential function of the form P(t) = a(b)^t, where t is yearssince 1900, which of the following would be a reasonable value for a?0.522.860.0351900

Answers

a represents the initial value of the function. Since the function starts at 1900, the associated population value for this year should be the answer. The answer is 0.52.

49. Calculate the limit. Some of these limits are made easier by considering the logarithm of the limit first, and some are not.

lim x^lnx
x→0+

Answers

The limit of xlnx when x is tending towards zero is zero.

What is limit in mathematics?

In mathematics, a limit is the value that a function (or sequence) approaches as the input (or index) approaches a particular value. Without limits, it is impossible to perform calculus or mathematical analysis; limits are also necessary to compute continuity, derivatives, and integrals.

Given limit is:

[tex]\lim_{x \to 0} x lnX[/tex]

Considering f = x, and g = ln x

The limit of f and g are both zero or both ±∞, and the limit f′/g′ exists, then the limit f/g equals it.

The wrong in the expression x lnx is we are individually defining f and g doesn't meet the hypothesis. so, we write it as

[tex]\frac{lnx}{1/x}[/tex]

We now use L' Hospital rule with f(x) = lnx, g(x) = 1/x as

The limits

[tex]\lim_{x \to 0^+} lnx= - \infty[/tex], and

[tex]\lim_{x \to 0^+} \frac{1}{x}[/tex]

The limits  [tex]\lim_{x \to 0^+} \frac{f'(x)}{g'(x)}[/tex] exists as:

[tex]\lim_{x \to 0^+} \frac{1/x}{-1/x^2} = \lim_{x \to 0^+} -x = 0[/tex]

Hence the answer is 0.

To know more about limits, go to link

https://brainly.com/question/23343679

#SPJ9

anyone know this one i was having troble

Answers

The zero of the linear function will be at (1, 0).

A linear function is the one which can be represented in the form y = ax + b where a, b are coefficients and x, and y are independent and dependent variables respectively. The zero of a line is the point where the line crossed the x-axis which is also known as the x intercept. The equation of the line in slope intercept form is given as y = mx + c where m is the slope and c is the y intercept. The slope of line from points (0, 5) and (2, -5) can be found by m = -5 - 5/2 -0

m = -10/2

m = -5

Now, putting this and the point (0, 5) in y = mx + c we get

5 = -5×0 + c

=> c = 5

Now, equation of line is y = -5x + 5 and for x intercept we put y = 0

0 = -5x + 5

5x = 5

=> x = 1

=> (1, 0) which is the required point

Learn more about linear function at:

brainly.com/question/4025726

#SPJ1

The slope of like A is m=9 & the slope of line B is m-9. What is their relation?
O Parallel
O Perpendicular
O Neither

Answers

the relationship is parallel

I need help on this question?

Answers

By definition, [tex](f \circ g)(x)=\boxed{f(g(x))}[/tex]. So, if [tex]g(2)=6[/tex] and [tex]f(6)=17[/tex], then [tex](f \circ g)(2)=f(g(2))=f(6)=\boxed{17}[/tex].

The state transportation commission counted cars traveling east and west across a toll bridge. 75% of the 588 cars counted were traveling east. How many eastbound cars did the commission count?

Answers

Answer: the answer is 441

Step-by-step explanation: 75% of 588 is 441

Maria expandió el siguiente cuadrado (x+3)^2=x^2 Está correcta la forma que usò?

Answers

No,  the form that Maria expanded is not correct.

What does it mean to "extend and simplify"?

We must multiply out the brackets in order to expand and simplify an expression, and we must then collect like words in order to simplify the resulting expression. We take out brackets by expanding them, often known as multiplying out.

Given:

(x+3)²

= x² + 2 . 3 . x + 3²

= x² + 6x + 9

Therefore the correct expansion is x² + 6x + 9.

To learn more about expansion click on the link below:

https://brainly.com/question/26430239

#SPJ9

Question:

Maria expanded the following square (x+3)^2=x^2 Is the form she used correct?

HELP ME WITH IS PROBLEM THANK YOU

Answers

The solution to the given expression is 10√5 - 4√10 which is the correct answer would be an option (C)

What are Arithmetic operations?

Arithmetic operations can also be specified by subtracting, dividing, and multiplying built-in functions. The operator that performs the arithmetic operation is called the arithmetic operator.

The given expression as:

⇒ √5(10 - 4√2)

Open the parentheses and apply the distributive property of multiplication to get,

⇒ 10√5 - 4√5√2

⇒ 10√5 - 4√(5 × 2)

⇒ 10√5 - 4√10

Therefore, the solution to the given expression is 10√5 - 4√10.

Learn more about Arithmetic operations here:

brainly.com/question/25834626

#SPJ1

Juan rented a truck for one day. There was a base fee of $9.00, and there was an additional charge of 7 cents for each mile driven. The total cost, C (in dollars), for driving x miles is given by the following function.
C (x) = 9.00 + 0.07x
What is the total rental cost if Juan drove 30 miles?

Answers

Answer:

$11.10

Step-by-step explanation:

Ez

I NEED HELP ASAP, I DONT KNOW HOW TO DO THIS.

Answers

Answer:

slope = 5/2

Step-by-step explanation:

Locations of points: (-4 , -4) and (-2 , 1)

slope = rise / run

between two plotted points this means that:

slope = change in y / change in x

slope = (1 - (-4)) / (-2 - (-4))

slope = 5/2

Solve and check the equation X-23=16

Answers

Answer:x=39

Step-by-step explanation:

x-23=16

x=16+23

x=39

a. If Ax = b has a solution and Aᵀy = 0, then y is perpendicular to ___.
b. If Aᵀy = c has a solution and Ax = 0, then x is perpendicular to ___.

Answers

The answer to question (a) that is if Ax = b has a solution and [tex]A^{T}[/tex] y = 0 then y is perpendicular to b

Answer to question (b) that is if [tex]A^{T}[/tex] y = c has a solution and Ax = 0 then x is perpendicular to c.

a) As Ax = b has a solution for x this means b lies in the column space of A. We also know that [tex]A^{T}[/tex] y = 0. This means y lies in the orthogonal complement of the column space of A or in the left null space of A. Therefore y is perpendicular to b

b) We see [tex]A^{T}[/tex] y = c has a solution. This tells us that c lies in the row space of A and also from the given Ax = 0 we can deduce that x lies in the null space of A or the orthogonal complement of row space of A. Therefore evidently  x will be perpendicular to c

Determine whether AT has a row space same as column space of A

//brainly.com/question/18994468

#SPJ4

Can you help me ?
This question has two parts. First, answer Part A. Then, answer Part B.
Part A
Fill in the blank question.
The cost of a school lunch is $2.50.



Complete the table to show the total cost of 1, 2, 3, and 4 lunches.



Lunches Bought 1 2 3 4
Total Cost ($)
Part B
Select the correct choices to complete the sentence.
The total cost is , 1 of 2.
Select Choice
to the number of lunches bought because the ratios between the quantities , 2 of 2.
Select Choice
the same unit rate.

Answers

The cost for the lunches will be:

1 lunch = $2.50

2 lunches = $5

3 lunches = $7.50

4 lunches = $10

How to calculate cost?

From the information, the cost of a school lunch is $2.50. Therefore, the cost of 1 lunch will be:

= 1 × $2.50

= $2.50

The cost of 2 lunches will be:

= 2 × $2.50

= $5.00

The cost of 3 lunches will be:

= 3 × $2.50

= $7.50

The cost of 4 lunches will be:

= 4 × $2.50

= $10

Learn more about cost on:

brainly.com/question/25109150

#SPJ1

Help please and thank you

Answers

On the x intercept of the graph, the y value is always 0. We can substitute y with 0. Therefore 4x-2(0)=-8. 4x=-8. x=-8/4= -2
Hence coordinates of x interfept = (-2,0)
On the y intercept, the x coordinate is 0. Hence 4(0) -2y = -8
-2y= -8
y=-8/-2 = 4
Y intercept coordinate is (0.4)

Which of the following would result in an integer?
O a.) 120
O b.) √144
O c.) √52
O d.) √24

Answers

The answer is either Option (a) or option(b) or both the options i.e, 120 or √144

What is Integer ?

An  integer is a whole number from the set of -ve , non-negative , +ve and 0 numbers.

Let's see how both the option is  correct

we know, integers are 0,1, 2, 3, .... or -1, -2 , -3 ......

Means the number which is not in fraction or decimal value or root.

so, by seeing option(a) we can say it's an integer

but in option(b) it's the root so how it is integer

The answer is simple it's perfect square of 12 so √144 = 12 so that means it have the value which is not in fraction. By that we can say that option(b) is also an integer

Now, in option(c) and (d) they are not the perfect square and their root results in decimal values. so we cannot tell them as an integer value.

Hence, the answer is either Option (a) or option(b) or both the options.

To read more about the Integers.

https://brainly.com/question/17695139

#SPJ13

In the diagram below, if < 2 = 123 °, what would be the measure of < 7?

Answers

Given:

[tex]\angle2=123^{\circ}[/tex]

To find:

The angle 7.

Explanation:

We know that,

The sum of the exterior angles is supplementary.

So, we can write

[tex]\begin{gathered} \angle2+\angle7=180 \\ 123^{\circ}+\angle7=180 \\ \angle7=180-123 \\ \angle7=57^{\circ} \end{gathered}[/tex]

Therefore, angle 7 is 57 degrees.

Final answer:

Angle 7 is 57 degrees.

From 1982 to 2004, the number
B
of federally insured banks could be approximated by
B
(
t
)
=

326.6
t
+
13783
where t is the year and t=0 corresponds to 1982.

How many federally insured banks were there in 1987?


Find the slope of the graph of B.

Answers

There were 120150 federally insured bank in the year 1987. The slope of the graph of B is -326.6.

The provided function B(x) states the number of federally insured banks from 1982-2004.

The function B(x) is,

B(x) = -326.6t + 13783

Here t is representing the year,

This function is a linear function corresponding to an equation of line,

If according to the situation,

t = 0, signifies the year 1982,

Then the year 1983 should be represented by t = 1.

On continuing the pattern the year 1987 should be represented by t = 5.

So, the number of federally insured banks in year 1987 can be found by putting t = 5 in B(x),

B(5) = -326.6(5) + 13783

B(5) = 12150.

The number of banks federally insured in year 1987 is 12150.

The function is B(x)= -326.6t + 1783,

This is representing the point-slope form of the line,

y = mx + c,

On comparing the provided function with the standard equation we can see,

Slope m = -326.6

So the slope of the function is -326.6.

To know more about linear functions, visit,

https://brainly.com/question/4025726

#SPJ1

LetW = the set of whole numbersF = the set of (non-negative) fractionsI = the set of integersN = the set of negative integersQ = the set of rational numbersSelect each set that is closed under addition.

Answers

Verify each set

W ---------> are closed under addition

F ------> are closed under addition

I ------> are closed under addition

N ----> are closed under addition

Q ----> are closed under addition

Hello I need help please.The choices for f(-6): A. f(-6) ; B. The answer is not a real number

Answers

Given the function:

[tex]f\mleft(x\mright)=\sqrt{x-1}[/tex]

First part:

You need to substitute the following value of "x" into the function:

[tex]x=1[/tex]

In order to find:

[tex]f(1)[/tex]

Then, substituting and evaluating, you get:

[tex]\begin{gathered} f(1)=\sqrt[]{1-1} \\ f(1)=\sqrt[]{0} \\ f(1)=0 \end{gathered}[/tex]

Second part:

Substitute this value of "x" into the function:

[tex]x=-6[/tex]

And then evaluate, in order to find:

[tex]f(-6)[/tex]

You get:

[tex]\begin{gathered} f(-6)=\sqrt[]{-6-1} \\ f(-6)=\sqrt[]{-7} \end{gathered}[/tex]

Since the Radicand is negative, it is not a Real Number.

Therefore, the answers are:

First part:

A.

[tex]f(1)=0[/tex]

Second part:

B. The answer is not a real number.

10. A 300-room hotel collects $125 per occupied room and does not collect any money for vacantrooms. Which of the following functions best represents how many dollars, d, the hotel generates ifthere are v vacant rooms in the hotel?O d = 125(300 + V)O d = 300(75 - v)O d = 300(75+)O d = 125(300 - V)

Answers

If there are v vacant rooms, the number of occupied rooms is 300-v (in total there are 300 rooms).

As every occupied makes the hotel collect 125, the dollars the hotel generates are:

[tex]d=125(300-v)[/tex]

It means that the correct answer is the last option.

Here are two expressions whose product is a new expression, A: 1. What could we put in the boxes to make A be a polynomial?

Answers

We are given the following product:

[tex](-2x{}^3+B)(x^n+15)=A[/tex]

For "A" to be a polynomial the value of the first box "B" must be a constant or a term of the form:

[tex]ax^n[/tex]

We will use a constant. We will substitute the first box for "1":

[tex](-2x^3+1)(x^n+15)=A[/tex]

For the second box, we need to use an exponent that is a positive whole number.

We will use 2:

[tex](-2x^3+1)(x^2+15)=A[/tex]

With these values, the result of the product is a polynomial.

Use the law of cosines to solve the following problem.The robot arm shown in the figure places packages on a conveyer belt. What is the distance x?

Answers

SOLUTION

Using cosine rule it follows:

[tex]x^2=3.00^2+2.25^2-3(2.25)\cos110[/tex]

Simplify the equation for x

[tex]x=\sqrt{3.00^2+2.25^2-3(2.25)\cos110}[/tex]

Therefore the required answer is

[tex]x=\sqrt{3.00^2+2.25^2-3(2.25)\cos(110)}[/tex]

Find the range PLEASE HELP!!! EXTRA POINTS

Answers

A is the answer because the graphing and if you put that into graph and measure points and look at function

the domain is the interval or set of all valid values for x (the input variable).

the range is the interval or set of all valid values for y or f(x) (the function result variable).

the functional result values for the first piece :

x - 4 for 0 <= x < 2

for x = 0 we get y = -4

for x = 2 we get y = -2

the range for this piece is [‐4, -2).

we use the round bracket to indicate -2 itself is not included, as we have x < 2 (and not x <= 2).

the functional result values for the second piece :

x² - 3x + 4 for 2 <= x < 4

for x = 2 we get 4-6+4 = 2

for x = 4 we get 16-12+4 = 8

the range for this piece is [2, 8)

again, because of x < 4 we use the ")" for 8 (not included).

the functional result values for the third piece :

5 for 4 <= x < 7

the range is simply [5) = [5] because the set contains only one element.

and 5 is already contained in the range of the second piece.

so, the range is just the combination of the ranges of the first 2 pieces :

[-4, -2) U [2, 8)

just 34 unless you want to help me with more?

Answers

M is the midpoint of the line segment FG.

Being the midpoint means that M divides the line in two equal halves so you know that the distance between F and M is equal to the distance between M and G, symbolically:

FM=MG

And of you add them you'll be able to calculate the length of the whole line FG

34.

Given the expressions

FM= 5y+13

MG=5-3y

We know that FM=MG, so the first step is to replace this equivalency with the given expressions. This way you'll determine an equation with one unknown y.

Once you have the equation you can clear the value of y:

[tex]\begin{gathered} FM=MG \\ 5y+13=5-3y \end{gathered}[/tex]

Pass all terms with the unknown to one side of the equation and all others to the other side:

[tex]\begin{gathered} 5y+3y=5-13 \\ 8y=-8 \\ y=-1 \end{gathered}[/tex]

y=-1

Now that you know the value of y, you can replace it in the given expressions to calculate the length of FG

[tex]\begin{gathered} FG=FM+MG \\ FG=(5y+13)+(5-3y) \\ FG=5y+13+5-3y \\ FG=2y+18 \\ \text{Replace with y=-1} \\ FG=2(-1)+18 \\ FG=16 \end{gathered}[/tex]

The length of the line segment FG is 16 units.

can you give me some examples of how to simplify

Answers

Let's check out the expression

[tex]2x+4+6y+7+2+3x[/tex]

To simplify, you have to add up similar terms. That is, add up everything that has an x, add up everything that has a y, and add up every independent term (no variable), as following:

[tex]\begin{gathered} 2x+4+6y+7+2+3x\rightarrow2x+3x+6y+4+7+2 \\ \rightarrow5x+6y+13 \end{gathered}[/tex]

And there we have the simplified expression.

Question 2-20
Mrs. Kimsey purchased a bushel of large apples that weighs about 48 pounds. The bushel contains 96 apples. What is the approximate
weight, in ounces, of one apple?
02
O 16
32

Answers

Weight of one apple is 8 ounces.

We have to calculate the weight of one apple. For that firstly, we have to calculate the weight of one apple in pounds because we have been given the weight of 96 apples in pounds. After that, we will convert it into ounces and we know that one pound is equal to 16 ounces.

A bushel has 96 apples.

We have been given weight of 1 bushel which is 48 pounds.

Weight of 96 apples = 48 pounds

We have to calculate weight of one apple.

Weight of one apple = 48/96 = 1/2 pound

1 pound = 16 ounces.

Now, we have to convert pounds into ounces

Weight of one apple = 1/2 * 16 ounces

= 8 ounces

Hence, the answer is 8 ounces.

To find more about how to calculate weight.

https://brainly.com/question/23245710

Given the graph above, what is the slope of this line?

Question 2 options:

1.m = -2


2. m = 3


3. m = 0


4. m = -3/2

Answers

Answer:

my answer would be m=-3/2

Step-by-step explanation:

given the coordinates

(0,0) - this is the origin and the first coordinate it will be treated as x1=0,y1=0

(-2,3)- this is the second given coordinate and will be treated as x2=-2,y2=3

To find the slope(m) we use the formula

m=y2-y1/x2-x1

m=3-0/-2-0

m=-(3/2)

m=-3/2

rate as brainliest

Other Questions
A woman has a total of $8,000 to invest. She invests part of the money in an account that pays 11 % per yearand the rest in an account that pays 12% per year. If the interest earned in the first year is $920, how much didshe invest in each account? What is 1 as a power of 10 Use the map below to answer the following question:Image of a map showing the migration of early humans. The map shows a migration path that begins in Africa and moves north, east, and south to Europe, Asia, and Australia. Another path is shown beginning in northern North America and moving southward through South America. Encyclopaedia Britannica, Inc. / Image Quest 2021To which continents did humans migrate last? Australia Asia South America Africa Henry is the fastest kid on the two legs. He won the semi-finals in sprinting. What is the best way to combine the two sentences above by using an appositive?A. The semi-finals in sprinting was won by the fastest kid on two legs. B. The fastest kid on two legs went to the semi-finals in sprintingC. Henry, the fastest kid on two legs, won the semi-finals in sprinting. If line q bisects KM at point V, KV = 2.5z,and VM = 5z - 10, find KM. No Errors can be syntax errors or logic errors (the code works, but not as intended). In the Guess My Number game, there is a lower limit and an upper limit for the range of possible numbers to guess. If the lower limit were exclusive and the upper limit inclusive, which expression would properly generate values for the secret number? (int)(Math.random() * (upper lower + 1) ) + lower (int)(Math.random() * (upper lower) ) + lower + 1 (int)(Math.random() * (upper lower + 1) ) + lower - 1 Write the inequality for the statement: the difference between a number and seven eighths exceeds negative two thirds f plus 7 over 8 is greater than 2 over 3 f minus 7 over 8 is less than or equal to negative 2 over 3 f minus 7 over 8 is greater than negative 2 over 3 f minus 7 over 8 is less than negative 2 over 3 PercentagesBand-aids are packaged in 100 per box. You had four full boxes when you started work yesterday. At the end ofthe day, you had two full boxes left. What polynomial identity should be used to prove that 61 = 125 64? aDifference of Cubes bDifference of Squares cSquare of a Binomial dSum of Cubes The narrators feelings towards the creature throughout the passage shift fromResponsesA frustration to reluctant acceptance.B jealousy to wild rage.C indifference to regretful sympathy.D anger to empathetic indecision. calculate the unemployment rate based on the following data for a country: population: 200, employed workers: 140, labor force: 160, discouraged workers: 10. select one: a. 10% b. 7.15% c. 12.5% d. 18.75% an investor purchases a treasury inflation protection securities (tips) bond with a 4% coupon. if during the first year the inflation rate is 9%, the approximate principal value of the security will be a politician who is running for the office of governor of a state with 4 million registered voters commissions a survey. in the survey, 54% of the 5,000 registered voters interviewed say they plan to vote for her. the population of interest is the: Lena is building a fence. She will need to dig up holes to help support the posts that hold up the fence. The holes need to have a depth of 3 1/3 feet below the ground. Each post is 10 feet long. What is the height of the part of the post that is above the ground? PLEASE HELP 100 points a flatbed truck is carrying a crate along a level road. the coefficient of static friction between the load and the bed is 0.40. the truck accelerates forward and the crate stays in its place on the truck bed. in what direction is the force that the bed exerts on the crate? what is the chance that among the three children in generation ii who have the dominant phenotype, one of them is aaaa and two of them are aaaa ? (hint: consider all possible orders of genotypes.) express your answer to two decimal places. If a high tide occurs at 6 A.M. and 6 P.M. At what time will the low tides occur? How much to the nearest penny will we have if he uses this investment? Round to the nearest cent if needed How to find the square root of [tex]\sqrt{28[/tex] How does your map illustrate the idea of Manifest Destiny? (at least a paragraph)